Pagina 1 di 2

Sarà irriducibile?

Inviato: 20 dic 2009, 11:42
da giove
Questo problema viene dalla gara della Mathesis di Brescia di quest'anno.

Determinare per quali interi positivi $ k $ il polinomio
$ a^{2k} + a^k b^k + b^{2k} $
è irriducibile negli interi.

Inviato: 20 dic 2009, 22:49
da Willy67
Cosa intendi per essere riducibile negli interi?

Inviato: 21 dic 2009, 00:33
da mantis
Un polinomio di questo tipo si può scrivere come $ (a^k+b^k)^2 - a^k b^k $.
Questa differenza con $ k = 2n $ diventa:
$ (a^{2n} + b^ {2n})^2 - a^ {2n} b^ {2n} = (a^ {2n} + b^ {2n} + a^n b^n)(a^{2n} + b^{2n} - a^n b^n) $. Quindi se $ k $ è pari il polinomio è riducibile.
Se $ k $ è dispari è chiaro che non è riducibile, poiché non si può applicare la differenza di quadrati.

Inviato: 21 dic 2009, 00:47
da Maioc92
mantis ha scritto: Se $ k $ è dispari è chiaro che non è riducibile, poiché non si può applicare la differenza di quadrati.
Questo è chiaro mi sembra alquanto poco chiaro....cosi non dimostri nulla, perchè questa non è una implicazione

Inviato: 21 dic 2009, 10:06
da mantis
Maioc92 ha scritto:
mantis ha scritto: Se $ k $ è dispari è chiaro che non è riducibile, poiché non si può applicare la differenza di quadrati.
Questo è chiaro mi sembra alquanto poco chiaro....cosi non dimostri nulla, perchè questa non è una implicazione
In effetti non è molto chiaro.. :oops:
Allora possiamo dire che se $ k = 2n + 1 $ si ha $ (a^{2n+1} + b^{2n+1})^2 - a^{2n+1} b^{2n+1} $ che porta a $ (a^{2n+1} + b^{2n+1} + a^{n+\frac{1}{2}} b^{n+\frac{1}{2}})(a^{2n+1} + b^{2n+1} - a^{n+\frac{1}{2}} b^{n+\frac{1}{2}}) $ che non è negli interi... va meglio?

Inviato: 21 dic 2009, 10:44
da Tibor Gallai
Va peggio.

E' come dire che 37 è un numero primo perché 37/2 fa 18,5 che non è intero.
E' vero che 37 è primo, ma dire che non è pari non basta per dimostrarlo.

Inviato: 21 dic 2009, 16:36
da giove
Willy67 ha scritto:Cosa intendi per essere riducibile negli interi?
Intendo che può essere scritto come prodotto di due polinomi in $ a $ e $ b $ a coefficienti interi.

Inviato: 21 dic 2009, 19:36
da Gauss91
E'

$ a^{2k} + a^k b^k + b^{2k} = \displaystyle\frac{a^{3k} - b^{3k}}{a^k - b^k} $.

Quindi è il "falso quadrato" di $ a^k-b^k $, che se non sbaglio è proprio irriducibile se l'esponente $ k $ è dispari. Sicuramente si può dim che non ha radici intere, ma non so se ciò basta per dire che è irriducibile. Se sì perché? Se no, perché?

Inviato: 21 dic 2009, 20:32
da giove
Gauss91 ha scritto:Se no, perché?
$ {x}^{5}+{x}^{4}+1 = \left( {x}^{2}+x+1\right) \,\left( {x}^{3}-x+1\right) $
Questo polinomio non ha radici intere ma è riducibile negli interi :)

Inviato: 22 dic 2009, 14:23
da Gabuntu94
Suggerimento:

$ x^2+x+1|x^{2n}+x^n+1 $ Per ogni n che non sia divisibile per tre :). Ora bisogna capire però quali sono quelli con n divisibile per tre che sono effettivamente non scomponibili...

Se serve posto la dimostrazione...

Inviato: 22 dic 2009, 15:24
da dario2994
Dato che per la prima volta da quando ho scoperto cosa sono sfrutto le radici dell'unità in una dimostrazione ci tengo a postarla xD Dimostro il lemmino ;)
Chiamo $ $P(x)=x^2+x+1 $ e $ $Q(x)=x^{2n}+x^n+1 $
Dimostro che P(x)|Q(x) se e solo se n non è divisibile per 3.
Chiamo $ $k,\bar{k} $ le radici di P(x) che usando la formula si rivelano essere complesse. È chiaro che se $ $Q(k)=0 $ allora vale anche $ $Q(\bar{k})=0 $ perchè altrimenti il polinomio non sarebbe a coefficienti reali. Noto che k è una radice terza dell'unità.
Dimostro: $ $k^{2n}+k^n+1=0 $ con n non divisibile per 3. Divido in casi:
3|n==> vale $ $Q(k)=k^{2\cdot 3z}+k^{3z}+1=1+1+1=3 $ perchè vale $ k^3=1 $
3|n+1: Sfruttando sempre che k è una radice terza dell'unità si riconduce a $ Q(k)=k^2+k+1 $ che vale 0 perchè è proprio P(x) di cui k è radice
3|n+2: Identico al caso precedente.

Spero di non aver toppato perchè mi piace particolarmente sta dimostrazione :) (magari ce n'è una molto più facile... anche se questa è davvero easy)

Inviato: 22 dic 2009, 15:28
da Gabuntu94
Ciao dario!

(Per quanto ne so io) E' giusta! Tra l'altro anche io l'ho fatto così! :D

In ogni caso si poteva anche scomporre a mano, ma era un po' più laborioso...



Questo però non risponde alla domanda di Giove... manca ancora un pezzo... :D

Inviato: 22 dic 2009, 15:50
da dario2994
Uhm... sto vicino alla conclusione...
Sono riuscito a dimostrare (con un metodo moooolto simile al precedente quindi lascio a chi vuole provare) che se chiamo m la valutazione 3-adica di n allora vale
$ $x^{2\cdot 3^m}+x^{3^m}+1|x^{2n}+x^n+1 $
Quindi rimane da dimostrare che $ $x^{2\cdot 3^k}+x^{3^k}+1 $ è irriducibile... ma non so se ci riesco :|
Spero di non aver sbagliato... comunque problema bellissimo (e ottimo per allenarsi ad usare le radici dell'unità)

Inviato: 22 dic 2009, 16:01
da Gabuntu94
Già... concordo!

Sono arrivato anche io al tuo stesso punto! Ma non riesco a dimostrare che se n è una potenza di tre allora il polinomio è EFFETTIVAMENTE irriducibile...

O meglio... ho quasi fatto, ma devo accertarmi che sia giusto! :D

Se ce la fai prima tu, fammi sapere tutto! :)

Inviato: 22 dic 2009, 18:18
da dario2994
Ho concluso... usando il teoremone xD
Praticamente $ $x^{2\cdot 3^k}+x^{3^k}+1 $ è il $ $3^{k+1}-esimo $ polinomio ciclotomico che per lemma noto è irriducibile negli interi.
Non so quanto sia difficile la dimostrazione del lemma... ma se è ad un livello accettabile sarebbe bello se qualcuno la postasse... o mettesse un link, sempre e solo se è capibile da uno con le mie conoscenze.

p.s. comunque bel problema... con una tesi che può tornare utile in un esercizio :)